Stay ahead of learning milestones! Enroll in a class over the summer!

G
Topic
First Poster
Last Poster
k a April Highlights and 2025 AoPS Online Class Information
jlacosta   0
Apr 2, 2025
Spring is in full swing and summer is right around the corner, what are your plans? At AoPS Online our schedule has new classes starting now through July, so be sure to keep your skills sharp and be prepared for the Fall school year! Check out the schedule of upcoming classes below.

WOOT early bird pricing is in effect, don’t miss out! If you took MathWOOT Level 2 last year, no worries, it is all new problems this year! Our Worldwide Online Olympiad Training program is for high school level competitors. AoPS designed these courses to help our top students get the deep focus they need to succeed in their specific competition goals. Check out the details at this link for all our WOOT programs in math, computer science, chemistry, and physics.

Looking for summer camps in math and language arts? Be sure to check out the video-based summer camps offered at the Virtual Campus that are 2- to 4-weeks in duration. There are middle and high school competition math camps as well as Math Beasts camps that review key topics coupled with fun explorations covering areas such as graph theory (Math Beasts Camp 6), cryptography (Math Beasts Camp 7-8), and topology (Math Beasts Camp 8-9)!

Be sure to mark your calendars for the following events:
[list][*]April 3rd (Webinar), 4pm PT/7:00pm ET, Learning with AoPS: Perspectives from a Parent, Math Camp Instructor, and University Professor
[*]April 8th (Math Jam), 4:30pm PT/7:30pm ET, 2025 MATHCOUNTS State Discussion
April 9th (Webinar), 4:00pm PT/7:00pm ET, Learn about Video-based Summer Camps at the Virtual Campus
[*]April 10th (Math Jam), 4:30pm PT/7:30pm ET, 2025 MathILy and MathILy-Er Math Jam: Multibackwards Numbers
[*]April 22nd (Webinar), 4:00pm PT/7:00pm ET, Competitive Programming at AoPS (USACO).[/list]
Our full course list for upcoming classes is below:
All classes run 7:30pm-8:45pm ET/4:30pm - 5:45pm PT unless otherwise noted.

Introductory: Grades 5-10

Prealgebra 1 Self-Paced

Prealgebra 1
Sunday, Apr 13 - Aug 10
Tuesday, May 13 - Aug 26
Thursday, May 29 - Sep 11
Sunday, Jun 15 - Oct 12
Monday, Jun 30 - Oct 20
Wednesday, Jul 16 - Oct 29

Prealgebra 2 Self-Paced

Prealgebra 2
Sunday, Apr 13 - Aug 10
Wednesday, May 7 - Aug 20
Monday, Jun 2 - Sep 22
Sunday, Jun 29 - Oct 26
Friday, Jul 25 - Nov 21

Introduction to Algebra A Self-Paced

Introduction to Algebra A
Monday, Apr 7 - Jul 28
Sunday, May 11 - Sep 14 (1:00 - 2:30 pm ET/10:00 - 11:30 am PT)
Wednesday, May 14 - Aug 27
Friday, May 30 - Sep 26
Monday, Jun 2 - Sep 22
Sunday, Jun 15 - Oct 12
Thursday, Jun 26 - Oct 9
Tuesday, Jul 15 - Oct 28

Introduction to Counting & Probability Self-Paced

Introduction to Counting & Probability
Wednesday, Apr 16 - Jul 2
Thursday, May 15 - Jul 31
Sunday, Jun 1 - Aug 24
Thursday, Jun 12 - Aug 28
Wednesday, Jul 9 - Sep 24
Sunday, Jul 27 - Oct 19

Introduction to Number Theory
Thursday, Apr 17 - Jul 3
Friday, May 9 - Aug 1
Wednesday, May 21 - Aug 6
Monday, Jun 9 - Aug 25
Sunday, Jun 15 - Sep 14
Tuesday, Jul 15 - Sep 30

Introduction to Algebra B Self-Paced

Introduction to Algebra B
Wednesday, Apr 16 - Jul 30
Tuesday, May 6 - Aug 19
Wednesday, Jun 4 - Sep 17
Sunday, Jun 22 - Oct 19
Friday, Jul 18 - Nov 14

Introduction to Geometry
Wednesday, Apr 23 - Oct 1
Sunday, May 11 - Nov 9
Tuesday, May 20 - Oct 28
Monday, Jun 16 - Dec 8
Friday, Jun 20 - Jan 9
Sunday, Jun 29 - Jan 11
Monday, Jul 14 - Jan 19

Intermediate: Grades 8-12

Intermediate Algebra
Monday, Apr 21 - Oct 13
Sunday, Jun 1 - Nov 23
Tuesday, Jun 10 - Nov 18
Wednesday, Jun 25 - Dec 10
Sunday, Jul 13 - Jan 18
Thursday, Jul 24 - Jan 22

Intermediate Counting & Probability
Wednesday, May 21 - Sep 17
Sunday, Jun 22 - Nov 2

Intermediate Number Theory
Friday, Apr 11 - Jun 27
Sunday, Jun 1 - Aug 24
Wednesday, Jun 18 - Sep 3

Precalculus
Wednesday, Apr 9 - Sep 3
Friday, May 16 - Oct 24
Sunday, Jun 1 - Nov 9
Monday, Jun 30 - Dec 8

Advanced: Grades 9-12

Olympiad Geometry
Tuesday, Jun 10 - Aug 26

Calculus
Tuesday, May 27 - Nov 11
Wednesday, Jun 25 - Dec 17

Group Theory
Thursday, Jun 12 - Sep 11

Contest Preparation: Grades 6-12

MATHCOUNTS/AMC 8 Basics
Wednesday, Apr 16 - Jul 2
Friday, May 23 - Aug 15
Monday, Jun 2 - Aug 18
Thursday, Jun 12 - Aug 28
Sunday, Jun 22 - Sep 21
Tues & Thurs, Jul 8 - Aug 14 (meets twice a week!)

MATHCOUNTS/AMC 8 Advanced
Friday, Apr 11 - Jun 27
Sunday, May 11 - Aug 10
Tuesday, May 27 - Aug 12
Wednesday, Jun 11 - Aug 27
Sunday, Jun 22 - Sep 21
Tues & Thurs, Jul 8 - Aug 14 (meets twice a week!)

AMC 10 Problem Series
Friday, May 9 - Aug 1
Sunday, Jun 1 - Aug 24
Thursday, Jun 12 - Aug 28
Tuesday, Jun 17 - Sep 2
Sunday, Jun 22 - Sep 21 (1:00 - 2:30 pm ET/10:00 - 11:30 am PT)
Monday, Jun 23 - Sep 15
Tues & Thurs, Jul 8 - Aug 14 (meets twice a week!)

AMC 10 Final Fives
Sunday, May 11 - Jun 8
Tuesday, May 27 - Jun 17
Monday, Jun 30 - Jul 21

AMC 12 Problem Series
Tuesday, May 27 - Aug 12
Thursday, Jun 12 - Aug 28
Sunday, Jun 22 - Sep 21
Wednesday, Aug 6 - Oct 22

AMC 12 Final Fives
Sunday, May 18 - Jun 15

F=ma Problem Series
Wednesday, Jun 11 - Aug 27

WOOT Programs
Visit the pages linked for full schedule details for each of these programs!


MathWOOT Level 1
MathWOOT Level 2
ChemWOOT
CodeWOOT
PhysicsWOOT

Programming

Introduction to Programming with Python
Thursday, May 22 - Aug 7
Sunday, Jun 15 - Sep 14 (1:00 - 2:30 pm ET/10:00 - 11:30 am PT)
Tuesday, Jun 17 - Sep 2
Monday, Jun 30 - Sep 22

Intermediate Programming with Python
Sunday, Jun 1 - Aug 24
Monday, Jun 30 - Sep 22

USACO Bronze Problem Series
Tuesday, May 13 - Jul 29
Sunday, Jun 22 - Sep 1

Physics

Introduction to Physics
Wednesday, May 21 - Aug 6
Sunday, Jun 15 - Sep 14
Monday, Jun 23 - Sep 15

Physics 1: Mechanics
Thursday, May 22 - Oct 30
Monday, Jun 23 - Dec 15

Relativity
Sat & Sun, Apr 26 - Apr 27 (4:00 - 7:00 pm ET/1:00 - 4:00pm PT)
Mon, Tue, Wed & Thurs, Jun 23 - Jun 26 (meets every day of the week!)
0 replies
jlacosta
Apr 2, 2025
0 replies
Hard Inequality Problem
Omerking   1
N 33 minutes ago by lpieleanu
$\frac{1}{a}+\frac{1}{b}+\frac{1}{c}=3$ is given where $a,b,c$ are positive reals. Prove that:
$$\frac{1}{\sqrt{a^3+1}}+\frac{1}{\sqrt{b^3+1}}+\frac{1}{\sqrt{c^3+1}} \le \frac{3}{\sqrt{2}}$$
1 reply
Omerking
Yesterday at 3:51 PM
lpieleanu
33 minutes ago
USAMO 2000 Problem 5
MithsApprentice   22
N 33 minutes ago by Maximilian113
Let $A_1A_2A_3$ be a triangle and let $\omega_1$ be a circle in its plane passing through $A_1$ and $A_2.$ Suppose there exist circles $\omega_2, \omega_3, \dots, \omega_7$ such that for $k = 2, 3, \dots, 7,$ $\omega_k$ is externally tangent to $\omega_{k-1}$ and passes through $A_k$ and $A_{k+1},$ where $A_{n+3} = A_{n}$ for all $n \ge 1$. Prove that $\omega_7 = \omega_1.$
22 replies
MithsApprentice
Oct 1, 2005
Maximilian113
33 minutes ago
f.e with finite number of f(t)=-t
jjkim0336   0
40 minutes ago
Source: own
f:R->R
f(xf(y)+y)=yf(x)+f(f(y)) and there are finite number of t such that f(t)= - t
0 replies
jjkim0336
40 minutes ago
0 replies
Common external tangents of two circles
a1267ab   55
N 41 minutes ago by awesomeming327.
Source: USA Winter TST for IMO 2020, Problem 2, by Merlijn Staps
Two circles $\Gamma_1$ and $\Gamma_2$ have common external tangents $\ell_1$ and $\ell_2$ meeting at $T$. Suppose $\ell_1$ touches $\Gamma_1$ at $A$ and $\ell_2$ touches $\Gamma_2$ at $B$. A circle $\Omega$ through $A$ and $B$ intersects $\Gamma_1$ again at $C$ and $\Gamma_2$ again at $D$, such that quadrilateral $ABCD$ is convex.

Suppose lines $AC$ and $BD$ meet at point $X$, while lines $AD$ and $BC$ meet at point $Y$. Show that $T$, $X$, $Y$ are collinear.

Merlijn Staps
55 replies
a1267ab
Dec 16, 2019
awesomeming327.
41 minutes ago
No more topics!
Angles summing to 180 equivalence [USA TST 2010 7]
MellowMelon   21
N Oct 14, 2024 by Eka01
In triangle ABC, let $P$ and $Q$ be two interior points such that $\angle ABP = \angle QBC$ and $\angle ACP = \angle QCB$. Point $D$ lies on segment $BC$. Prove that $\angle APB + \angle DPC = 180^\circ$ if and only if $\angle AQC + \angle DQB = 180^\circ$.
21 replies
MellowMelon
Jul 26, 2010
Eka01
Oct 14, 2024
Angles summing to 180 equivalence [USA TST 2010 7]
G H J
The post below has been deleted. Click to close.
This post has been deleted. Click here to see post.
MellowMelon
5850 posts
#1 • 3 Y
Y by Adventure10, Mango247, Rounak_iitr
In triangle ABC, let $P$ and $Q$ be two interior points such that $\angle ABP = \angle QBC$ and $\angle ACP = \angle QCB$. Point $D$ lies on segment $BC$. Prove that $\angle APB + \angle DPC = 180^\circ$ if and only if $\angle AQC + \angle DQB = 180^\circ$.
Z K Y
The post below has been deleted. Click to close.
This post has been deleted. Click here to see post.
Luis González
4148 posts
#2 • 2 Y
Y by Adventure10, Mango247
$P,Q$ are obviously isogonal conjugates WRT $\triangle ABC.$ Let $AP,AQ$ cut $BC$ at $N,M.$ Since $\angle BPN =180^{\circ}-\angle APB$ and $\angle CQM=180^{\circ}-\angle AQC,$ the problem is equivalent to prove that there exists a point $D$ on $\overline{BC}$ such that $PN,PD$ are isogonal WRT $\angle BPC$ and $QM,QD$ are isogonal WRT $\angle BQC.$ Assume there exists such a $D.$ By Steiner theorem we have:

$ \frac{BN}{NC} \cdot \frac{BD}{DC}=\frac{BP^2}{CP^2} \ , \ \frac{BM}{MC} \cdot \frac{BD}{DC}=\frac{BQ^2}{CQ^2} \ \Longrightarrow$

$\frac{BQ^2}{CQ^2} \cdot \frac{MC}{BM}=\frac{BP^2}{CP^2} \cdot \frac{NC}{BN} \Longrightarrow \ \frac{BQ}{CQ} \cdot \frac{\sin \widehat{AQC}}{\sin \widehat{AQB}}=\frac{BP}{CP} \cdot \frac{ \sin \widehat{APC}}{\sin \widehat{APB}}$

$\Longrightarrow \ \frac{ \sin \widehat{AQC}}{\sin \widehat{QCA}} \cdot \frac{\sin \widehat{QBA}}{\sin \widehat{AQB}}=\frac{\sin \widehat{APC}}{\sin \widehat{PCA}} \cdot \frac{\sin \widehat{PBA}}{\sin \widehat{APB}}$

$\Longrightarrow \ \frac{AC}{AQ} \cdot \frac{AQ}{AB}=\frac{AC}{AP} \cdot \frac{AP}{AB}, $ which is obvious identity. Thus, there exists such a $D.$
Z K Y
The post below has been deleted. Click to close.
This post has been deleted. Click here to see post.
math154
4302 posts
#3 • 4 Y
Y by Adventure10, Mango247, and 2 other users
Obviously $P,Q$ are isogonal conjugates. Since the second condition is equivalent to $\angle{AQB}+\angle{DQC}=180^\circ$, which is analogous to the first condition (i.e. $P,Q$ are switched), it suffices to prove that the first condition implies the second. By the Law of Sines on $\triangle{DPB},\triangle{DPC}$ in addition to the angle conditions, we have
\[\frac{BD}{\sin{APC}}=\frac{BD}{\sin(180^\circ-DPB)}=\frac{BD}{\sin{DPB}}=\frac{DP}{\sin{PBC}},\\
\frac{CD}{\sin{APB}}=\frac{CD}{\sin(180^\circ-DPC)}=\frac{CD}{\sin{DPC}}=\frac{DP}{\sin{PCB}},\]so
\[\frac{BD\sin{APB}}{CD\sin{APC}}=\frac{\sin{PCB}}{\sin{PBC}}\qquad(1).\]Now by the Law of Sines on $\triangle{AQB},\triangle{AQC}$ and isogonal conjugates $P,Q$,
\[\frac{AB}{\sin{AQB}}=\frac{AQ}{\sin{QBA}}=\frac{AQ}{\sin{PBC}}\\
\frac{AC}{\sin{AQC}}=\frac{AQ}{\sin{QCA}}=\frac{AQ}{\sin{PCB}},\]so from (1),
\[\frac{AB\sin{AQC}}{AC\sin{AQB}}=\frac{\sin{PCB}}{\sin{PBC}}=\frac{BD\sin{APB}}{CD\sin{APC}}\qquad(2).\]But again by the Law of Sines on $\triangle{DQB},\triangle{DQC}$ and isogonal conjugates $P,Q$, we have
\[\frac{BD}{\sin{DQB}}=\frac{DQ}{\sin{QBC}}=\frac{DQ}{\sin{PBA}},\\
\frac{CD}{\sin{DQC}}=\frac{DQ}{\sin{QCB}}=\frac{DQ}{\sin{PCA}},\]so dividing and applying the Law of Sines on $\triangle{PCA},\triangle{PBA}$,
\[\frac{BD\sin{DQC}}{CD\sin{DQB}}=\frac{\sin{PCA}}{\sin{PBA}}=\frac{\frac{AP}{AC}\sin{APC}}{\frac{AP}{AB}\sin{APB}}=\frac{AB\sin{APC}}{AC\sin{APB}}.\]Multiplying this equation with (2), we obtain
\[\sin{DQC}\sin{AQC}=\sin{DQB}\sin{AQB},\]or (note that $(DQC+AQC)=360^\circ-(DQB+AQB)$)
\begin{align*}
\cos(DQC-AQC)-\cos(DQC+AQC)&=\cos(DQB-AQB)-\cos(DQB+AQB)\\
\implies\cos(DQC-AQC)&=\cos(DQB-AQB).\end{align*}Clearly angles $DQC,AQC,DQB,AQB$ are all in $(0,180^\circ)$, so their differences are in $(-180^\circ,180^\circ)$. Thus we either have
\[DQC-AQC=DQB-AQB \implies DQC+AQB=DQB+AQC=180^\circ,\]as desired, or
\[DQC-AQC=-DQB+AQB \implies DQC+DQB=AQC+AQB=180^\circ,\]contradicting the fact that $Q$ is inside $\triangle{ABC}$.
Z K Y
The post below has been deleted. Click to close.
This post has been deleted. Click here to see post.
abacadaea
2176 posts
#4 • 1 Y
Y by Adventure10
Click to reveal hidden text
Z K Y
The post below has been deleted. Click to close.
This post has been deleted. Click here to see post.
Zhero
2043 posts
#5 • 3 Y
Y by Adventure10, Mango247, and 1 other user
All angles in this proof are directed.

Lemma 1: Let $Q$ be an interior point of $\triangle ABC$, and let $D$, $E$, and $F$ be the projections of $Q$ onto $BC$, $CA$, and $AB$, respectively. Let $O$ be the circumcenter of $\triangle DEF$, and let $M$ be the midpoint of $OA$. Then $\angle MOF = \angle ACQ + \angle QBA.$
Proof: $FE$ is a common chord of the circumcircles of $AFQE$ and $\triangle DEF$, so $MO \perp EF$. Hence,
\[ \angle MOF \&= \frac{\angle EOF}{2} \&= \angle EDF = \angle EDQ + \angle QDF \&= \angle ACQ + \angle QBA. \]
Corollary: Let $P'$ be the reflection of $Q$ across $O$, and let $P$ be the isogonal conjugate of $Q$. Then $P' = P$.
Proof: Let $M$ and $N$ be the midpoints of $AQ$ and $BQ$, respectively, and let $F$ be the projection from $Q$ onto $AB$. By the lemma,
\begin{align*}
\angle MON 
&= \angle MOF + \angle NOF = (\angle ACQ + \angle QBA) + (\angle QCB + \angle BAQ) \\
&= (\angle PCB + \angle PBC) + (\angle PCA + \angle CAP) \\
&= (180^{\circ} - \angle BPC) + (180^{\circ} - \angle CPA) \\
&= \angle APB.
\end{align*}

A homothety centered at $Q$ with factor 2 reveals that $\angle AP'B = \angle APB$. Similarly, we find that $\angle CP'A = \angle CPA$ and $\angle BP'C = \angle BPC$. Hence, $P'$ lies on the intersection of the circumcircles of $\triangle ABP$, $\triangle CPA$, and $\triangle BPC$. But these circumcircles intersect only at the point $P$, so we have $P = P'$, as desired.


Let $R$ and $S$ be the reflections of $P$ and $Q$ across $BC$, respectively, and let $D' = PS \cap QR$. Because $D'$ lies on the axis of symmetry of isosceles trapezoid $PRSQ$, $D'$ must lie on $BC$ as well. As $D$ varies from $C$ to $B$, $\angle DPC$ increases and $\angle DQB$ decreases, so there exists at most one point $D$ such that $\angle APB + \angle DPC = 180^{\circ}$ and $\angle AQC + \angle DQB = 180^{\circ}$. We claim that $D = D'$ satisfies both of these conditions.

By symmetry, it is sufficient to show that $\angle APB + \angle D'PC = 180^{\circ}$. Let $O$ be the the midpoint of $PQ$, let $M$ be the midpoint of $CQ$, and let $T$ be the midpoint of $QS$. By the corollary to our lemma, $O$ is the circumcenter of the $Q$-pedal triangle of $\triangle ABC$, so by the lemma, $\angle TOM =  \angle QCA + \angle ABQ = 180^{\circ} - \angle APB$. A homothety centered at $Q$ with factor 2 sends $M$ to $C$, $O$ to $P$, and $T$ to $S$, so $\angle TOM = \angle SPC = \angle D'PC = 180^{\circ} - \angle APB$, which completes our proof.

Motivation
Z K Y
The post below has been deleted. Click to close.
This post has been deleted. Click here to see post.
SnowEverywhere
801 posts
#6 • 6 Y
Y by baladin, R8450932, tapir1729, magicarrow, Adventure10, and 1 other user
Angle chasing yields that the isogonal conjugate of $A$ with respect to $\triangle{BPC}$ is the reflection $Q'$ of $Q$ in line $BC$. The condition $\angle APB+\angle DPC = 180^\circ$ is therefore equivalent to $P$, $Q'$ and $D$ being collinear. Similarly, the condition $\angle AQC+\angle DQB = 180^\circ$ is equivalent to $Q$, $P'$ and $D$ being collienear where $P'$ is the reflection of $P$ in $BC$. Now note that quadrilaterals $BPCQ'$ and $BP'CQ$ are reflections in $BC$. Therefore $PQ'$ and $P'Q$ divide $BC$ in the same ratio and concur on $BC$. This implies the result.
Z K Y
The post below has been deleted. Click to close.
This post has been deleted. Click here to see post.
Sardor
804 posts
#7 • 1 Y
Y by Adventure10
Let $ AP \cap BC=S $ and $ AQ \cap BC=T $ and $ \angle BAP=\angle CAQ=x, \angle CBP=\angle ABQ=y $ and $ \angle ACQ=\angle BCP=z $.After Steiner's theorem , this problem is equivalent to $ \frac{BP^2}{CP^2} \cdot \frac{BS}{SC}=\frac{BQ^2}{CQ^2} \cdot \frac{BT}{ST} (*) $.By sine law to the triangles $ BPC $ and $ BQC $, $ (*) $ is equivalent to $ ( \frac{sinx}{sin(A-x)} \cdot  \frac{siny}{sin(B-y)} \cdot  \frac{sinz}{sin(C-z)})^2=1 $, which is true by Ceva's theorem.
This post has been edited 2 times. Last edited by Sardor, Dec 16, 2014, 11:23 AM
Z K Y
The post below has been deleted. Click to close.
This post has been deleted. Click here to see post.
TelvCohl
2312 posts
#8 • 3 Y
Y by baladin, Adventure10, and 1 other user
My solution:

Lemma :

Let $ P,P' $ be the isogonal conjugate of $ \triangle ABC $ .
Let $ \ell $ be the isogonal conjugate of $ AP $ of $ \angle BPC $ .
Let $ \ell ' $ be the isogonal conjugate of $ AP' $ of $ \angle BP'C $ .
Then $ \ell, \ell' $ are symmetry WRT $ BC $.

Proof of the lemma:

Let $ X $ be the intersection of $ BP' $ and $ CP $ .
Let $ Y $ lie on $ BC $ satisfy $ XA,XY $ are isogonal conjugate of $ \angle BXC $ .

Since $ CA,CY $ are isogonal conjugate of $ \angle P'CP $ ,
so $ A,Y $ are isogonal conjugate of $ \triangle CXP' $ .
ie. $ P'Y,P'A $ are isogonal conjugate of $ \angle BP'C $ .
Similarly, we can prove $ PY, PA $ are isogonal conjugate of $ \angle BPC $ .

Since $ \angle PYB=\angle CYP' $ (Easy angle chasing) ,
so we get $ PY,P'Y $ are symmetry WRT $ BC $ .

Back to the main problem :

From lemma we get

$ \angle APB+\angle DPC=180^{\circ}  \Longleftrightarrow PA, PD $ are isogonal conjugate of $ \angle BPC $

$ \Longleftrightarrow QA, QD $ are isogonal conjugate of $ \angle BQC \Longleftrightarrow \angle AQC+\angle DQB=180^{\circ} $

Q.E.D
Z K Y
The post below has been deleted. Click to close.
This post has been deleted. Click here to see post.
Dukejukem
695 posts
#9 • 2 Y
Y by Adventure10, Mango247
Let $\triangle A'B'C'$ be the pedal triangle of $Q$ WRT $\triangle ABC$ and let $\triangle Q_aQ_bQ_c$ be the triangle obtained by reflecting $Q$ in $BC, CA, AB.$ Since $P, Q$ are isogonal conjugates, it is well-known (Corollary to Fact 3) that $P$ is the circumcenter of $\triangle Q_aQ_bQ_c.$ Therefore $BP$ is the perpendicular bisector of $\overline{Q_aQ_c}$, implying that \[\measuredangle BQ_aP = -\measuredangle Q_aPB - \measuredangle PBQ_a = -\measuredangle Q_aPB - \measuredangle ABC = -\measuredangle Q_aQ_bQ_c - \measuredangle ABC.\]Furthermore, since $Q, A, B', C'$ are inscribed in the circle of diameter $\overline{AQ}$, with similar relations holding for the two other sets of vertices, we have \[\measuredangle Q_aQ_bQ_c = \measuredangle A'B'C' = \measuredangle A'B'Q + \measuredangle QB'C' = \measuredangle A'CQ + \measuredangle QAC' = \measuredangle BCQ + \measuredangle QAB = -\measuredangle CQA - \measuredangle ABC,\]where the last step follows from examining quadrilateral $ABCQ.$ Consequently, if $Q_aP$ cuts $BC$ at $D'$, then \[\measuredangle BQD' = -\measuredangle BQ_aD' = -\measuredangle BQ_aP = -\measuredangle CQA,\]implying that $D' \equiv D.$ Thus, if $P_a$ is the reflection of $P$ in $BC$, then by symmetry in trapezoid $QQ_aP_aP$, we have $D \in QP_a$, and analagous arguments show that $\measuredangle CPD = \measuredangle APB.$ The desired result follows. $\square$
Z K Y
The post below has been deleted. Click to close.
This post has been deleted. Click here to see post.
v_Enhance
6876 posts
#10 • 8 Y
Y by rkm0959, Ankoganit, magicarrow, lahmacun, v4913, jeteagle, Adventure10, Mango247
For uniqueness reasons it suffices to show that there is a single point $D$ satisfying both conditions at once. To construct it, take the ellipse with foci $P$ and $Q$ tangent to the sides $BC$, $CA$, $AB$ at $D$, $E$, $F$. We claim this works.

It's a known property of ellipses that $\overline{CP}$ bisects $\angle DPE$. Similarly, $\overline{BP}$ bisects $\angle FPD$ and $\overline{AP}$ bisects $\angle EPF$. Thus $\angle APB + \angle DQC = 180^{\circ}$ follows, and in the same way $\angle AQC + \angle DQB = 180^{\circ}$.
Z K Y
The post below has been deleted. Click to close.
This post has been deleted. Click here to see post.
menpo
209 posts
#11
Y by
My solution
Lemma: If the points $P^\ast, Q^\ast $ are symmetric to the points $ P, Q $ with respect to the line $ BC $, then $ P^\ast$ and $ A $ are isogonal conjugates with respect to $\triangle BQC,$ and also $Q^\ast$ and $ A $ are isogonal conjugates with respect to $\triangle BPC.$

[asy]
import olympiad;
size(12cm);
defaultpen(fontsize(10pt));

pair A = dir(120);
pair B = dir(210);
pair C = dir(330);
pair X = (7A+2C)/(7+2);
pair Y = (5A+3B)/(5+3);
pair P = extension(B,X,C,Y);
pair P0 = foot(P,B,C);
pair Ps = 2*P0 - P;
pair X0 = foot(P,A,C);
pair S = 2*X0 - P;
pair Y0 = foot(P,A,B);
pair K = 2*Y0 - P;
pair Q = circumcenter(Ps,S,K);
pair Q0 = foot(Q,B,C);
pair Qs = 2*Q0 - Q;
pair D = extension(P,Qs,Q,Ps);
pair T = (8P-5A)/(8-5);
pair Z = (5Q-1.5*A)/(5-1.5);


draw(A--T, dotted);
draw(A--Z, dotted);
draw(A--B--C--A,red);
draw(B--P--C, black);
draw(B--Q--C, black);
draw(Q--Ps, dashed);
draw(P--Qs, dashed);
draw(B--Ps, black);
draw(C--Ps, black);
draw(B--Qs, black);
draw(C--Qs, black);
draw(anglemark(Qs,B,C), blue);
draw(anglemark(D,B,Q), blue);
draw(anglemark(P,B,A), blue);

draw(anglemark(A,C,P), darkgreen);
draw(anglemark(Q,C,D), darkgreen);
draw(anglemark(D,C,Qs), darkgreen);

dot("$A$", A, dir(A));
dot("$B$", B, dir(B));
dot("$C$", C, dir(C));
dot("$Q$", Q, dir(120));
dot("$P$", P, dir(120));
dot("$P^\ast$", Ps, dir(Ps));
dot("$Q^\ast$", Qs, dir(Qs));
dot("$D$", D, dir(380));
[/asy]

Note that,
$$\measuredangle PBA=\measuredangle CBQ=\measuredangle Q^{\ast}BC,$$$$\measuredangle PCA=\measuredangle BCQ=\measuredangle Q^{\ast}CB,$$therefore $P^\ast$ and $ A $ are isogonal conjugates with respect to $\triangle BQC.$ Similarly, $ Q^\ast $ and $A$ are isogonal conjugates with respect to $\triangle BPC.\blacksquare $

Therefore, the lines $PA$ and $ PQ^{\ast} $ are isogonal conjugates relative to $\angle BPC $, as well as the lines $ QA $ and $ QP^{\ast} $ are isogonal conjugates relative to $\angle BQC, $ whence $$ \angle APB + \angle Q^{\ast}PC = 180^{\circ}, $$$$ \angle AQC + \angle P^{\ast}QB = 180^{\circ}, $$Then it suffices to prove that $ D $ lies on the ray $ QP^{\ast} $ if and only if $D$ lies on the ray $ PQ^{\ast}, $ which is obvious, since $ QP^{\ast} $ and $ PQ^{\ast}$ intersect on $ BC. $
This post has been edited 3 times. Last edited by menpo, Dec 1, 2020, 10:03 AM
Z K Y
The post below has been deleted. Click to close.
This post has been deleted. Click here to see post.
GeronimoStilton
1521 posts
#12 • 3 Y
Y by Mango247, Mango247, Mango247
Observe that the point $D_1$ such that $\angle APB+\angle D_1PC=180^\circ$ must be such that $PD_1$ and $AP$ are isogonal conjugates relative to $\angle BPC$. Let $AP$ intersect line $BC$ at $X$, then this result implies
\[\frac{BX}{XC}\cdot\frac{BD_1}{D_1C}=\frac{BP^2}{PC^2}.\]Then if we define $Y$ as the intersection of lines $AQ$ and $BC$, it suffices to check
\[\frac{BP^2}{PC^2}\cdot\frac{XC}{XB}=\frac{BQ^2}{QC^2}\cdot\frac{YC}{YB}\iff \frac{BP^2}{PC^2}\cdot\frac{XC^2}{XB^2}=\frac{BQ^2}{QC^2}\cdot\frac{YC}{YB}\cdot\frac{XC}{XB}=\frac{BQ^2}{QC^2}\cdot\frac{AC^2}{AB^2}.\]So it is equivalent to check
\[1=\frac{BP\cdot XC\cdot QC\cdot AB}{BQ\cdot AC\cdot PC\cdot XB}=\frac{AP\cdot \frac{\sin\angle BAP}{\sin\angle ABP}\cdot AC\cdot\frac{\sin\angle PAC}{\sin\angle AXC}\cdot AQ\cdot\frac{\sin\angle QAC}{\sin\angle QCA}\cdot AB}{AQ\cdot\frac{\sin\angle BAQ}{\sin\angle ABQ}\cdot AC\cdot AP\cdot\frac{\sin\angle PAC}{\sin\angle ACP}\cdot AB\cdot\frac{\sin\angle PAB}{\sin\angle AXB}}=\]\[\frac{\frac{\sin\angle BAP}{\sin\angle ABP}\cdot \frac{1}{\sin\angle PCB}}{\frac{\sin\angle PAC}{\sin\angle PBC}\cdot \frac{1}{\sin\angle ACP}}=\frac{\sin\angle BAP}{\sin\angle ABP}\cdot\frac{\sin\angle PBC}{\sin\angle PCB}\cdot\frac{\sin\angle ACP}{\sin\angle PAC}=\frac{BP}{AP}\cdot\frac{PC}{PB}\cdot\frac{AP}{CP},\]which is trivial.
Z K Y
The post below has been deleted. Click to close.
This post has been deleted. Click here to see post.
jeteagle
480 posts
#13
Y by
First, we have a theorem (Isogonal Ratios Theorem): If $AX$ and $AY$ are isogonal with respect to $\angle{BAC}$ and $X, Y \in BC$, then $$\frac{BD\cdot BE}{CD\cdot CE} = \left(\frac{AB}{AC}\right)^2$$
We prove the if version because the only if part is symmetrical. Notice that $P$ and $Q$ are isogonal conjugates. Let $E$ and $F$ be the intersections of $AP$ and $AQ$ with $BC$ respectively. By our theorem, we have the two equations: $$\frac{BE\cdot BF}{CE\cdot CF} = \left(\frac{AB}{AC}\right)^2 \implies \frac{BF}{CF} = \left(\frac{AB}{AC}\right)^2\cdot \frac{CE}{BE}$$$$\frac{BE\cdot BD}{CE\cdot CD} = \left(\frac{BP}{CP}\right)^2 \implies \frac{BD}{CD} = \left(\frac{BP}{CP}\right)^2\cdot \frac{CE}{BE}$$and we wish to prove $$\left(\frac{BQ}{CQ}\right)^2 = \frac{BD\cdot BF}{CD\cdot CF} = \left(\frac{AB\cdot BP\cdot CE}{AC\cdot CP\cdot BE}\right)^2 \implies 1 = \frac{AB\cdot BP\cdot CE\cdot CQ}{AC\cdot CP\cdot BE\cdot BQ}.$$
Now, notice by Law of Sines that $\frac{AB}{BE} = \frac{\sin{\angle{AEB}}}{\sin{\angle{BAE}}}$ and $\frac{AC}{CE} = \frac{\sin{\angle{AEC}}}{\sin{\angle{CAE}}} = \frac{\sin{\angle{AEB}}}{\sin{\angle{CAE}}}$ so $$\frac{AB\cdot BP\cdot CE\cdot CQ}{AC\cdot CP\cdot BE\cdot BQ} = \frac{BP\cdot CQ}{CP\cdot BQ}\cdot \frac{\sin{\angle{CAE}}}{\sin{\angle{BAE}}}.$$
Additionally, $\frac{BP}{CP} = \frac{\sin{\angle{BCP}}}{\sin{\angle{CBP}}}$ and $\frac{CQ}{BQ} = \frac{\sin{\angle{CBQ}}}{\sin{\angle{BCQ}}}$. Therefore, we have $$\frac{BP\cdot CQ}{CP\cdot BQ}\cdot \frac{\sin{\angle{CAE}}}{\sin{\angle{BAE}}} = \frac{\sin{\angle{BCP}}\cdot\sin{\angle{CBQ}}\cdot\sin{\angle{CAE}}}{\sin{\angle{CBP}}\cdot\sin{\angle{BCQ}}\cdot\sin{\angle{BAE}}}$$$$= \frac{\sin{\angle{BCP}}\cdot\sin{\angle{ABP}}\cdot\sin{\angle{CAP}}}{\sin{\angle{CBP}}\cdot\sin{\angle{ACP}}\cdot\sin{\angle{BAP}}} = 1$$by Trig Ceva's, as desired. $\blacksquare$
Z K Y
The post below has been deleted. Click to close.
This post has been deleted. Click here to see post.
ike.chen
1162 posts
#14
Y by
WLOG, assume $\angle APB + \angle DPC = 180^{\circ}$. In addition, we define $X = AP \cap BC$ and $Y = AQ \cap BC$. It's clear that $P$ and $Q$ are isogonal conjugates wrt $ABC$.

Notice $$\angle BPX = 180^{\circ} - \angle APB = \angle DPC$$which implies $PX$ and $PY$ are isogonal in $\angle BPC$. Thus, applying Steiner's Ratio Theorem twice yields $$\frac{AB^2}{AC^2} = \frac{BX}{XC} \cdot \frac{BY}{YC}$$and $$\frac{PB^2}{PC^2} = \frac{BX}{XC} \cdot \frac{BD}{DC}.$$Now, the LoS and Ratio Lemma give $$\frac{QB^2}{QC^2} = \left( \frac{\sin BAQ \cdot \frac{AQ}{\sin ABQ}}{\sin CAQ \cdot \frac{AQ}{\sin ACQ}} \right)^2 = \left( \frac{\sin BAY}{\sin CAY} \cdot \frac{\sin ACQ}{\sin ABQ} \right)^2$$$$= \left(\frac{BY}{YC} \cdot \frac{AC}{AB} \cdot \frac{\sin PCB}{\sin PBC} \right)^2 = \left(\frac{BY}{YC} \cdot \frac{AC}{AB} \right)^2 \cdot \frac{PB^2}{PC^2}$$$$= \left(\frac{AB}{AC} \cdot \frac{XC}{BX} \right)^2 \cdot \frac{BX}{XC} \cdot \frac{BD}{DC} = \frac{AB^2}{AC^2} \cdot \frac{XC}{BX} \cdot \frac{BD}{DC} = \frac{BY}{YC} \cdot \frac{BD}{DC}.$$Hence, the Converse of Steiner's implies $\angle BQD = \angle YQC$, which clearly finishes. $\blacksquare$


Remarks: This problem should be pretty straightforward, as utilizing Steiner's is the obvious approach. We can also reverse engineer certain ratios to help us simplify our trigonometric expression.
Z K Y
The post below has been deleted. Click to close.
This post has been deleted. Click here to see post.
MathsLover04
95 posts
#15
Y by
Clearly, $P$ and $Q$ are isogonal conjugates. So if we denote $AP\cap BC=\{X\}$ and $AQ\cap BC=\{Y\}$, we have that $\angle BAY=\angle CAX$.
So the conclusion would be equivalent with \[\left(\frac{QB}{QC}\right)^2\cdot \frac{YC}{YB}=\left(\frac{BP}{PC}\right)^2\cdot \frac{XC}{XB}\]which reduces, after using the sine law in the triangles $\Delta BPA$, $\Delta BQA$, $\Delta CPA$ and $\Delta QCA$, to \[\frac{\sin{\angle QCA}}{\sin{\angle QCB}}\cdot \frac{\sin{\angle QBC}}{\sin{\angle{QBA}}}\cdot \frac{\sin{\angle BAQ}}{\sin{\angle{CAQ}}}=1\]which is true, by Ceva.
Z K Y
The post below has been deleted. Click to close.
This post has been deleted. Click here to see post.
Cookierookie
52 posts
#16 • 2 Y
Y by Mango247, Mango247
Lemma: Point $P$ inside quadrilateral $ABCD$ has an isogonal conjugate iff $\angle APB + \angle CPD = 180$.
Proof: Construct the pedal quadrilateral of P, it's not hard to show that both conditions are equivalent to the pedal quadrilateral being concyclic.

Applying this lemma on degenerate quadrilateral $ABDC$ kills the problem.
Z K Y
The post below has been deleted. Click to close.
This post has been deleted. Click here to see post.
asdf334
7585 posts
#17
Y by
here lies dumb solution (good solution hopefully coming soon.)

Let $AP\cap BC=X$ and let $AQ\cap BC=Y$. It suffices to show that
\[\left(\frac{PB}{PC}\right)^2\div \frac{XB}{XC}=\left(\frac{QB}{QC}\right)^2\div \frac{YB}{YC}\]or that
\[\frac{PB}{PC}\div \frac{QB}{QC}=\sqrt{\frac{XB}{XC}\div \frac{YB}{YC}}\]
Now use Ratio Lemma on $\triangle ABC$ with $AX$, $AY$ as cevians. After some annoying calculations we can find
\[\frac{XB}{XC}\div \frac{YB}{YC}=\frac{\sin^2 \angle BAP}{\sin^2 \angle CAP}\]and LoS on $\triangle ABP$ and $\triangle ACP$ gives
\[\sin BAP=\frac{PB\cdot \sin \angle ABP}{PA}\]and dividing gives
\[\frac{XB}{XC}\div \frac{YB}{YC}=\left(\frac{PB\cdot QC}{PC\cdot QB}\right)^2\]and we're done.
Z K Y
The post below has been deleted. Click to close.
This post has been deleted. Click here to see post.
Shreyasharma
680 posts
#18
Y by
Obviously $P$ and $Q$ are isogonal conjugates. Then we may consider the inellipse with foci $P$ and $Q$, inscribed in $\triangle A BC$and denote by $X$, $Y$ and $Z$ its tangency points to $\overline{BC}$, $\overline{CA}$ and $\overline{AB}$.

Claim: The point $X$ is the unique point along $\overline{BC}$ satisfying $\angle APB + \angle XPC = 180$.
Proof. The uniqueness part is not hard to see. To show the angle equality note that,
\begin{align*}
\angle APB + \angle XPC &= (\angle APZ + \angle ZPB) + \angle XPC\\
&= \angle APY + \angle BPZ + \angle CPX\\
&= \frac{1}{2}(\angle YPZ + \angle ZPX + \angle XPY)\\
&= \frac{1}{2}(360) = 180
\end{align*}proving the claim. $\square$

Then an identical angle chase for $Q$ proves the claim and we're done.
Z K Y
The post below has been deleted. Click to close.
This post has been deleted. Click here to see post.
dolphinday
1325 posts
#19 • 1 Y
Y by ehuseyinyigit
Clearly $P$ and $Q$ are isogonal conjugates in $\triangle ABC$. The condition $\angle APB + \angle DPC = 180^\circ$ implies that $P$ has an isogonal conjugate wrt degenerate quadrilateral $ABDC$, which is clearly $Q$ as $\angle ACB = \angle ACD$ and $\angle ABC = \angle ABD$. Clearly the reverse holds true as well, so we are done.

This problem also follows from the fact that the inellipse with foci $P$ and $Q$ is tangent to sides $BD$ and $CD$ of deg. quadrilateral $ABDC$, so $P$ and $Q$ are isogonal conjugates in $ABCD$.
Z K Y
The post below has been deleted. Click to close.
This post has been deleted. Click here to see post.
Clew28
45 posts
#20 • 1 Y
Y by duckman234
Given that \( P \) and \( Q \) are isogonal conjugates with respect to \( \triangle ABC \), we know that \( AP \) and \( AQ \) are isogonal lines, meaning the angles they form with the triangle's sides are equal when measured from corresponding vertices. If \( N \) is the intersection of \( AP \) with \( BC \) and \( M \) is the intersection of \( AQ \) with \( BC \), we aim to prove the existence of a point \( D \) on \( BC \) such that \( PN \) and \( PD \) are isogonal with respect to \( \angle BPC \) and \( QM \) and \( QD \) are isogonal with respect to \( \angle BQC \).

To approach this problem, we will use the properties of trigonometric Ceva's theorem and the ratios resulting from the sine rule. First, applying Steiner's theorem to the triangles formed by \( P \), \( Q \), and segments on \( BC \), and specifically using the points \( N \), \( M \), and \( D \), we get the following relationships:

\[
\frac{BN}{NC} \cdot \frac{BD}{DC} = \frac{BP^2}{CP^2}
\]\[
\frac{BM}{MC} \cdot \frac{BD}{DC} = \frac{BQ^2}{CQ^2}
\]
By equating the product of these ratios, we find:

\[
\frac{BQ^2}{CQ^2} \cdot \frac{MC}{BM} = \frac{BP^2}{CP^2} \cdot \frac{NC}{BN}
\]
Using the sine rule in triangles \( \triangle BPA \), \( \triangle BQA \), \( \triangle CPA \), and \( \triangle QCA \), we can express these ratios in terms of sines of angles, giving us:

\[
\frac{BQ}{CQ} \cdot \frac{\sin \angle AQC}{\sin \angle AQB} = \frac{BP}{CP} \cdot \frac{\sin \angle APC}{\sin \angle APB}
\]
From the isogonal property, we have:

\[
\frac{\sin \angle AQC}{\sin \angle QCA} \cdot \frac{\sin \angle QBA}{\sin \angle AQB} = \frac{\sin \angle APC}{\sin \angle PCA} \cdot \frac{\sin \angle PBA}{\sin \angle APB}
\]
Simplifying this using the fact that \( P \) and \( Q \) are isogonal conjugates, we obtain:

\[
\frac{AC}{AQ} \cdot \frac{AQ}{AB} = \frac{AC}{AP} \cdot \frac{AP}{AB}
\]
This is an obvious identity as both sides simplify to 1. Thus, there exists a point \( D \) on \( \overline{BC} \) that satisfies the required conditions. By verifying the conditions, we conclude that \( PN \) and \( PD \) are isogonal with respect to \( \angle BPC \), and \( QM \) and \( QD \) are isogonal with respect to \( \angle BQC \). Therefore, the existence of such a point \( D \) is guaranteed by the properties of isogonal conjugates and the applications of trigonometric Ceva's theorem.
Z K Y
The post below has been deleted. Click to close.
This post has been deleted. Click here to see post.
Aiden-1089
279 posts
#21 • 1 Y
Y by ehuseyinyigit
Note that $P$ and $Q$ are isogonal conjugates wrt $\Delta ABC$. So $\angle APB + \angle DPC = 180^\circ \iff P$ and $Q$ are isogonal conjugates in $ABDC \iff \angle AQC + \angle DQB = 180^\circ$.
Z K Y
The post below has been deleted. Click to close.
This post has been deleted. Click here to see post.
Eka01
204 posts
#22
Y by
The given angle conditions imply that $P$ and $Q$ are isogonal conjugates in $\Delta ABC$. Then $\angle APB +\angle CPD=180^\circ$ implies $P$ has an isogonal conjugate in the degenerate quadrilateral $ABDC$ so it must in fact be $Q$ so the desired result follows. Same goes for the other direction as they are both symmetric.
Z K Y
N Quick Reply
G
H
=
a